What is the slope of the line whose equation is y-4=5/2(x-2)?

What Is The Slope Of The Line Whose Equation Is Y-4=5/2(x-2)?

Answers

Answer 1

Answer:

[tex]slope = \frac{ - 1 - 4}{0 - 2} \\ = \frac{ - 5}{ - 2} \\ = { \tt{ \frac{5}{2} }}[/tex]


Related Questions

c) Which of these numbers is a square number?
A
4 x 105
B
9 x 104
С
4 x 103
D
9 x 103

Answers

The answer is B 9 x 10^5
Because the root of it is 300

Find the height h of the triangle.

Answers

Answer:

A, 12 times the square root of 3

Step-by-step explanation:

24 sqaured plus 12 squared=432

square root of 432=20.9

12 times the sqaure root of 3=20.9

What is the value of x?

Answers

Answer:

D. 30°

Step-by-step explanation:

By exterior angle theorem:

x° + 70° = 100°

x° = 100° - 70°

x° = 30°

x = 30

find the slope of the line passing through the points (-2,5) and (3/2,2)

Answers

Answer:

slope = - [tex]\frac{6}{7}[/tex]

Step-by-step explanation:

Calculate the slope m using the slope formula

m = [tex]\frac{y_{2}-y_{1} }{x_{2}-x_{1} }[/tex]

with (x₁, y₁ ) = (- 2, 5) and (x₂, y₂ ) = ([tex]\frac{3}{2}[/tex], 2)

m = [tex]\frac{2-5}{\frac{3}{2}-(-2) }[/tex]

   = [tex]\frac{-3}{\frac{3}{2}+2 }[/tex]

   = [tex]\frac{-3}{\frac{7}{2} }[/tex]

  = - 3  × [tex]\frac{2}{7}[/tex]

  = - [tex]\frac{6}{7}[/tex]

5) On each birthday Rosa gets as many roses as she is old in years. She still has all the dried flowers and there are now 120 of them. How old is she? A) 10 B) 12 C) 14 D) 15 E) 20

Answers

Answer:

D) 15

Step-by-step explanation:

This is an arithmatic progression.

The formula for the sum of arithmatic progression is

[tex]s = \frac{n}{2} (2a + (n - 1)d)[/tex]

where d is the common difference between successive terms and a is the first term. By applying this formula,

[tex]120 = \frac{n}{2} (2(1) + (n - 1)(1)) \\ 120 = \frac{n}{2} (1 + n) \\ n(1 + n) = 240 \\ n {}^{2} + n - 240 = 0 \\ (n - 15)(n + 16) = 0 \\ n = 15 \: or \: n = - 16(reject)[/tex]

Consider the graph of f(x) = 5x + 1. Explain how to find the average rate of change between x = 0 and x = 4.
What is the average rate of change?

Answers

Answer:

5

Step-by-step explanation:

You divide the change in the output value by the change in the input value.

  Input: 0 |  4

Output: 1  |  21

20/4= 5

At a certain store, a CD costs $12. If the cost of CDs were graphed as the output, compared to the number of
CDs purchased as input, which of the following would not be true of the graph?
A. The set of points would all lie on the same line.
B. The set of points would include the origin.
C. The set of points would rise from left to right.
D. The set of points would not graph a function.
Please select the best answer from the choices provided
A
B
C
D

Answers

Answer: The best and most correct answer among the choices provided by the question is A. The set of points would all lie on the same line.    

     

Need help~ What is the measure of arc sty in the circle O below

Answers

Answer:

50

Step-by-step explanation:

Arc AT the center = angle at the center = 50

Heya Kitties!
What is the value of x?

18x−16=−12x−4



x =

Answers

Answer:

2/5

Step-by-step explanation:

18x+12x=-4+16

30x=12

x=2/5

Answer:

18x-16=-12x-4

18x+12x=-4+16

30x=12

x=12/30

x=2/5

ax + by = c and mx + ny = d and an # bm then these simultaneous equations have a) Only one common solution. b) No solution. c) Infinite number of solutions. d) Only two solutions.​

Answers

Answer:

a) Only one common solution

Step-by-step explanation:

The first line has slope of a/b and the second one has slope of m/n.

As an ≠ bm ⇒ a.b ≠ m/n, the slopes are different.

Since the slopes are different the lines are not parallel, hence they intersect at one point.

This means there is one solution only.

ax + by = c and mx + ny = d and an # bm then these simultaneous equations have Only one common solution.

[tex]\sf{ }[/tex] [tex]\sf{ }[/tex]

the area of a circle is 616 m square find the radius 5 equals to 22/7 ​

Answers

Step-by-step explanation:

[tex]here \: is \: your \: solution : - \\ \\ GIVEN \: \: -:- \\ \\ = > \: area \: of \: circle \: = 616 \: m {}^{2} \\ \\ \ = > pi = 22 \div 7 \\ \\ = > we \: need \: to \: find \: radius \: \\ \\ = > area \: of \: circle \: = \pi \: r {}^{2} \\ \\ = > \: \pi \: r {}^{2} = 616 \: m {}^{2} \\ \\ = > \: r {}^{2} \times (22 \div 7) = 616 \\ \\ = > \: r {}^{2} =( 616 \times 7) \div 22 \\ \\ = > \: r { }^{2} = 4312 \div 22 \\ \\ = > r {}^{2} = 196 \\ \\ = > \: r = \sqrt{196} \\ \\ = > \: r = 14 \: \: \: (ANSWER✓✓✓) \\ \\ HOPE \: IT \: HELPS \: YOU \: (◕ᴗ◕✿)[/tex]

On the unit circle, which of the following angles has the terminal point
coordinates.
A. 45
B. 135
C. 225
D. 315

Answers

Answer: C. 225

Step-by-step explanation:

Question
The quotient of a number and 5 has a result of 2. What is the number?

Answers

Answer:

10.

Step-by-step explanation:

Answer:

The number is 10.

Step-by-step explanation:

x/5 = 2

Multiply both sides by 5.

5 * x/5 = 5 * 2

x = 10

Answer: The number is 10.

2m^2-5m-3=0 by factorization​

Answers

Answer:

M= 6, -1

Step-by-step explanation:

Factoring these numbers, it will result in (m-6)(m+1). So, m= 6,-1

Select the expression that represents the following statement: add 24 to the quotient of 16 and 8.

Answers

Answer:

16/8 + 24

Step-by-step explanation:

which describes the graph of y=-(x+5)^2+3

Answers

Answer:

Maximum at (-5, 3)

Step-by-step explanation:

The quadratic equation is in vertex form, which specifically shows the vertex.

Since the a is negative, you get a maximum y value of 3.

Vertex form: y=a(x-h)^2+k

Maximum at (-5, 3)
Y’ = -2(x+5)
=> maximum when y’ = 0

Helppp and explain pls and thankyou

Answers

Answer:

None of both

Step-by-step explanation:

It say y = lxl

so it mean if y = 3 so x = 3 or -3

Someone tell me where everyone is going right please !!

Answers

Answer:

1. 3rd option

2. 4th option

3. 1st option

4. 2nd option

5. 2nd option

Step-by-step explanation:

1.

on changing the signs of an equality the sign gets reveresd

-6 < -2x

6 > 2x

3 > x

2.

2x - 3 > 11 - 5x

adding 5x and 3 to both sides and combining like terms

(2x + 5x) + (-3 + 3) > (11 + 3) + (-5x + 5x)

7x > 14

x > 2

put of all the options x = 4 is a value greater than 2 thus satisfying the condition.

3.

6k + 10. 5 = 3k + 12

subtracting 3k and 10.5 from both the sides while combining like terms

(6k - 3k) + (10.5 - 10.5) = (3k - 3k) + (12 - 10.5)

3k = 1.5

k = 0.5

4.

y + 3 = -y + 9

adding y and subtracting 3 from both sides

(y + y) + (3 - 3) = (-y + y) + (9 - 3)

2y = 6

y = 3

5.

-9x + 1 = -x + 17

adding x and subtracting 1 from both sides

(-9x + x) + (1 -1) = (-x +x) + (17 -1)

-8x = 16

multiplying the equation by -1

8x = -16

diving both sides by 8

x = -2

Answer:

[tex]1. \: \: x < 3[/tex]

[tex]2. \: \: 2[/tex]

[tex]3. \: \: k = 0.5[/tex]

[tex]4. \: \: y = 3[/tex]

[tex]5. \: \: x = 12[/tex]

Step-by-step explanation:

[tex]1. \: \: - 2(5 - 4x) < 6x - 4 \\ - 10 + 8x < 6x - 4 \\ - 10 + 2x < - 4 \\ 2x < 6 \\ x < 3[/tex]

[tex]2. \: \: 2x - 3 > 11 - 5x \\ 7x - 3 > 11 \\ 7x > 14 \\ x = 2[/tex]

[tex]3. \: \: 6k + 10.5 = 3k + 12 \\ 3k + 10.5 = 12 \\ 3k = 1.5 \\ k = 0.5[/tex]

[tex]4. \: \: y + 3 = - y + 9 \\ 2y + 3 = 9 \\ 2y = 6 \\ y = 3[/tex]

[tex]5. \: \: - 9x + 1 = - x + 17 \\ - 8x + 1 = 17 \\ - 8x = 16 \\ x = - 2[/tex]

Hope it is helpful....

What is the value of k?

Answers

Answer:

Step-by-step explanation:

recall that the inside angles of a triangle add up to 180°

and also that the angle 115° is the other part of a 180° total

180=115+y

180-115 =65

then the inside angle at Y  is 65°

add the three inside (interior of the triangle) angles up

180= 65 + (4k+5) + (6k+10)

180 = 65 + 10k + 15

180 = 80 + 10k

100  = 10k

10 = k

see how I did that?  

Nathan is 1.55 meters tall. At 1 p.m., he measures the length of a tree's shadow to be 38.15 meters. He stands 32.9 meters away from the tree, so that the tip of his shadow meets the tip of the tree's shadow. Find the height of the tree to the nearest hundredth of a meter .

Answers

Answer:

11.26 m

Step-by-step explanation:

The height of the tree is about 11.25 meters.

What are similar triangles?

When the respective sides are proportional and the corresponding angles are congruent, two triangles are said to be similar.

Given that, the height of the person is 1.55 meters, the length of the tree's shadow is 38.15 meters, and the distance between the person and the tree is 32.9 meters.

Let the height of the tree be x.

Note that the scenario makes two similar triangles.

Since the ratio of the side lengths of similar triangles is proportional, it follows:

(38.15 - 32.9)/1.55 = 38.15/x

5.25/1.55 = 38.15/x

3.39 = 38.15/x

x = 38.15/3.39

x = 11.25

Hence, the height of the tree is about 11.25 meters.

Learn more about similar triangles: https://brainly.com/question/25882965

#SPJ2

According to the number line, what is the distance between points A and B?

0 6 units
7 units
O 12 units
O 14 units

Answers

Answer:

14 units

Step-by-step explanation:

A = - 2, B = 12

Therefore,

d(A, B) = 12 - (-2) = 12 + 2 = 14 units

How do u solve this ?

Answers

You want to distribute the x

jordan wants to save to buy a car and decides to open a banking account that is offering an interest rate of 4.5% compounded annually how much will jordan have in the account after 5 years it he deposits $7,000 today?

Answers

Answer:

8,723.27$

Step-by-step explanation:

Hello, Brainly community!

This question is for all of those Calculus people out there.

The volume of a swimming pool is changing with respect to time, such that the volume is given by W(t), where W(t) is measured in cubic centimeters and t is measured in seconds. A tangent line is shown for W(t) at t = 3 seconds. Determine the best estimate for the value of the instantaneous rate of change of W(t) when t = 3.
(I've narrowed down the answer choices to 2, and just really need to find the right way of thinking to find the answer)

(A) W(lim t) as t goes to 3.
(B) [W(3.1) - W(2.9)] / 0.2.

Thank you in advance!

Answers

Answer:

(B)  [tex]\displaystyle \frac{W(3.1) - W(2.9)}{0.2}[/tex]

General Formulas and Concepts:

Calculus

Limits

Derivatives

The definition of a derivative is the slope of the tangent line.

Derivative Notation

Instantaneous Rates

Tangent Line: [tex]\displaystyle f'(x) = \frac{f(b) - f(a)}{b - a}[/tex]

Step-by-step explanation:

Since we are trying to find a rate at which W(t) changes, we must find the derivative at t = 3.

We are given 2 close answer choices that would have the same numerical answer but different meanings:

(A)  [tex]\displaystyle \lim_{t \to 3} W(t)[/tex](B)  [tex]\displaystyle \frac{W(3.1) - W(2.9)}{0.2}[/tex]

If we look at answer choice (A), we see that our units would simply just be volume. It would not have the units of a rate of change. Yes, it may be the closest numerically correct answer, but it does not tell us the rate at which the volume would be changing and it is not a derivative.

If we look at answer choice (B), we see that our units would be cm³/s, and that is most certainly a rate of change. Answer choice (B) is also a derivative at t = 3, and a derivative tells us what rate something is changing.

∴ Answer choice (B) will give us the best estimate for the value of the instantaneous rate of change of W(t) when t = 3.

Topic: AP Calculus AB/BC (Calculus I/I + II)

Unit: Differentiation

Book: College Calculus 10e

A cinema is doing a promotion to celebrate their 50th anniversary for 1 week. They give

away a free drink to every 98th customer, a free bag of popcorn to every 112th customer and

a free cinema ticket to every 224th customer. Which lucky customer will be the first to

receive all 3 items?​

Answers

Answer:

1,568 customer

Step-by-step explanation:

Find the lowest common multiple of 98, 112, and 224

98 = 98, 196, 294, 392, 490, 588, 686, 784, 882, 980, 1078, 1176, 1274, 1372, 1470, 1568, 1666

112 = 112, 224, 336, 448, 560, 672, 784, 896, 1008, 1120, 1232, 1344, 1456, 1568, 1680, 1792, 1904

224 = 224, 448, 672, 896, 1120, 1344, 1568, 1792, 2016, 2240

The lowest common multiple of 98, 112, and 224 is 1568

Therefore, the 1,568th customer will be the first to receive all 3 iitem

What type of health screening would this patient most likely receive?
Sue is a 45-year-old woman with a family history of breast cancer. Her healthcare professional will most likely recommend that she receive a .

Answers

Answer:

A mammogram is what she would receive

Step-by-step explanation:

How is mathematical thinking established? How can we have mathematical thinking?

修改翻译结果

Answers

All I see are the two questions and either mandarine or Japanese

Translate How is mathematical thinking established? How can we have mathematical thinking? in to Chinese.

数学思维是如何建立的? 怎样才能有数学思维?

Solve for x. Round to the nearest tenth, if necessary.

Answers

Answer:

X would be 63.9

Hope it helps

Step-by-step explanation:

The value of the variable 'x' using the cosine formula will be 63.9 units.

What is a right-angle triangle?

It's a form of a triangle with one 90-degree angle that follows Pythagoras' theorem and can be solved using the trigonometry function.

Trigonometric functions examine the interaction between the dimensions and angles of a triangular form.

The value of 'x' is given by the cosine of the angle ∠QSR. And the cosine of an angle is the ratio of the base and hypotenuse of the right-angle triangle. Then we have

cos 35° = x / 78

x = 63.9

The value of the variable 'x' using the cosine formula will be 63.9 units.

More about the right-angle triangle link is given below.

https://brainly.com/question/3770177

#SPJ7

find the domain of f(x)=sec(2x)

Answers

Answer:

*Refer the image attached

Step-by-step explanation:

*Refer the image attached

Plot point C in GeoGebra to verify that it indeed lies on AB. You can enter the coordinates in the Input window to plot the point . Also , verify the distances you calculated in question 3 using GeoGebra . Take a screenshot showing the distances displayed by GeoGebra , and paste it below .

Answers

Answer:

The picture of file

Step-by-step explanation:

Your'e welcome

Other Questions
how did nationalism contribute to global conflict following world war 1 Which three of the following conditions on Earth at the end of the Ice Age made human survival easier?abundance of large animalsdecrease in sea levelsextinction of mammothsincrease in vegetationwarmer global climate 5 positive integers are arranged in ascending order, as follows:1,9, 9, 10, XThe mean and the median are equal.Find X. A car at a position 150 km [W] of Toronto travels to a position 400 km [W] ofToronto. The total displacement of the car is: Acetic acid and water react to form hydronium cation and acetate anion, like this: HCH3CO2(aq) + H2O H3O+(l)(aq) + CH3CO2^-(aq) Imagine 226 mmol of CH3CO2- are added to a flask containing a mixture of HCH3CO2, H2O, H3O + and CH3CO2- at equilibrium.Required:What is the rate of the reverse reaction before any CH3CO2^- has t:een added to the flask? Consider a binomial experiment. If the number of trials is increased, what happens to the expected value stuck on this problem The most common carrier of communicable diseases are a.cockroach b.housefly c.mosquito d.spider Managing your resources requires:A spending what your earn, saving only when you have money left over, eating convenient fast food,B paying bills and taxes on time, staying healthy, and spending less than you earn.C limiting yourself on food, clothing, and entertainment.D saving 50% of your earnings and becoming a minimalist. in the outsiders which line best supports the authors message that some things in life are out of people control? conclusion de la Moral During the Industrial Revolution, textile factories (blank)-produced items which increased the available options and lowered the prices for consumers. What is true of mechanoreceptors? A) They sense lightB) They sense heat/coldC) They respond to pressure, movement, and tension.D) They monitor painE) Other: a right triangle has a leg of length 13 and a hypotnuse of length 41 yards. find the length of the other leg (nearest tenth) Please answer the two questions below. please help so I can watch Coop and Cami ask the world Need help on last question Solve the given system by the substitution method. 3x + y = 8 7x - 4y = 6 A random sample of 35 employees of the local green technologies plant Greenies, who completed two years of college, were asked to take a basic mathematics test. The mean and standard deviation of their scores were 75.1 and 12.8, respectively. In a random sample of 50 employees who had only completed high school, the mean and standard deviation of the test scores were 72.1 and 14.6, respectively. Assuming equal variance between the two populations, can we infer at the .10 level of significance that students who completed two years of college had a higher average than students who had only completed high school How much carbon dioxide is released when it is fully combusted with 4Kg of ethanol with more than enough oxygen? How do you work it out?